LSAT and Law School Admissions Forum

Get expert LSAT preparation and law school admissions advice from PowerScore Test Preparation.

 Administrator
PowerScore Staff
  • PowerScore Staff
  • Posts: 8917
  • Joined: Feb 02, 2011
|
#40574
Complete Question Explanation
(The complete setup for this game can be found here: lsat/viewtopic.php?t=5718)

The correct answer choice is (A)

If S and R are shown together on the same screen, it must be on screen 1 or 2, which both have two showings. However, from the third rule it cannot be screen 2, and thus we can infer that S and R are shown together on screen 1. The remaining three variables are H, M, and W, which must then be shown on screens 2 and 3:
PT70 -Game_#3_#18_diagram 1.png
From the fourth rule H and M cannot be shown on the same screen, and thus we can infer that one of H and M is shown on screen 3, and the remainder is shown on screen 2 with W:
PT70 -Game_#3_#18_diagram 2.png
With the groups established, the linear information can now be applied.

On screen 1, S and R have no restrictions, and can rotate freely. On screen 2, W cannot be shown at 9 P.M. due to the first rule, and thus W must be shown at 7 P.M., with H or M shown at 9 P.M. On screen 3, either H or M could be shown at 8 P.M.:
PT70 -Game_#3_#18_diagram 3.png
Accordingly, answer choice (A) must be true and is therefore correct.
You do not have the required permissions to view the files attached to this post.

Get the most out of your LSAT Prep Plus subscription.

Analyze and track your performance with our Testing and Analytics Package.